Beziehung zwischen Propagator und Wahrscheinlichkeit für den unendlichen Brunnen

Dies mag eine einfache Frage sein, aber ich bin wirklich verwirrt darüber.

Für den unendlichen quadratischen Brunnen sind die (zeitabhängigen) Energieeigenfunktionen (innerhalb des Brunnens):

ψ N ( X , T ) = 2 / L e ich E N T / S ich N ( N π L X )

mit E N = N 2 π 2 2 M L 2 2 die Eigenwerte der Energie und L , die Breite des Brunnens. Also die Wahrscheinlichkeit, ein Energieteilchen zu finden E M zwischen X = A Und X = B zum Zeitpunkt T ist durch die Bornsche Regel gegeben:

P ( A , B ; T ) = A B ψ M ( X ) ψ M ( X ) D X

Diese Wahrscheinlichkeit könnte als die Wahrscheinlichkeit verstanden werden, dass ein Teilchen dazwischen gefunden wird A Und B irgendwann und jeden | ψ M ( X ) ψ M ( X ) | 2 ist eine quadrierte Wahrscheinlichkeit .

Andererseits ist der Propagator die Amplitude, mit der sich das Teilchen fortbewegt A B rechtzeitig T A T B :

P R Ö P A G A T Ö R = X B , T B | X A , T A = X B , T B | ( M | M M | ) | X A , T A = M ψ M ( X B , T B ) ψ M ( X A , T A ) = M e ich E M ( T B T A ) / ψ M ( X B ) ψ M ( X A )

Meine Frage ist also: Wenn der Propagator eine Amplitude ist, sollte das Quadrieren eine Wahrscheinlichkeit ergeben. Quadrieren von Gleichung (3):

P A B = | X B , T B | X A , T A | 2 = | M e ich E M ( T B T A ) / ψ M ( X B ) ψ M ( X A ) | 2

was offensichtlich keine Wahrscheinlichkeit ist, da ein Mitglied gefällt | ψ M ( X ) ψ M ( X ) | 2 ist eigentlich eine quadrierte Wahrscheinlichkeit . Wie bekomme ich dann eine Wahrscheinlichkeit vom Propagator?

Antworten (2)

was offensichtlich keine Wahrscheinlichkeit ist, da ein Mitglied gefällt | ψ M ( X ) ψ M ( X ) | 2 ist eigentlich eine quadrierte Wahrscheinlichkeit. Wie bekomme ich dann eine Wahrscheinlichkeit vom Propagator?

Richtig, aber nur eines Ihrer x ist eine Variable und trägt zur Wahrscheinlichkeitsamplitude bei. Erinnern Sie sich daran, dass Ihr Anfangszustand ist | X 0 , T 0 , wenn wir also in Eigenzustände expandieren wollen, schreiben wir (erlauben Sie mir zu nehmen T 0 = 0 )

| Ψ ( X 0 , 0 ) = | X 0 , T 0 = M | M M | X 0 , T 0

Wir können den Endzustand in ähnlicher Weise in Eigenzustände erweitern

| Ψ ( X , T ) = | X , T = N e ich E N T | N N | X , T

so dass, wenn wir die Überlappung nehmen, die Summen auf eins zusammenbrechen

X , T | X 0 , T 0 = M e ich E M T / X , T | M M | X 0 , T 0 = X , T | M e ich E M T / | M M | X 0 , T 0

Der Punkt ist, dass die "zusätzlichen" Terme nicht Teil Ihrer Dichte sind, sie sind in Bezug auf die endgültige Position konstant.


Seien Sie auch vorsichtig. Dies wird nicht die Wahrscheinlichkeit geben. Was die Wahrscheinlichkeit geben wird, ist

P ( A X B ) = A B | X , T | X 0 , T 0 | 2 D X

Ich denke, das OP hat in einem dimensionalen Sinne nach den "zusätzlichen" Begriffen gefragt. Insbesondere seit ψ M ( X ) ψ M ( X 0 ) hat bereits die Dimension einer Wahrscheinlichkeitsdichte (egal ob X 0 eine Konstante ist oder nicht), hätte sein Modulusquadrat problematische Dimensionen, um eine Wahrscheinlichkeitsdichte zu sein. Entschuldigung, wenn ich entweder OP oder Sie falsch verstanden habe.
Vielen Dank für die tolle Antwort. Alles ist klarer, aber das Dimensionsproblem, auf das Dvij Mankad gerade hingewiesen hat. Weißt du, wie man es repariert?
Ich denke, da die End- und Anfangszustände Positionseigenzustände sind, wirft das die Dimensionen ab. Ich denke, das könnte etwas helfen . physical.stackexchange.com/questions/185962/…
Ich sehe es nicht so deutlich. Wie es bei einigen Antworten auf die von Ihnen zitierte Frage geschrieben steht, wenn | X X | D X = 1 , dann brauchen wir die Maße von | X sein 1 / L . Also, noch einmal, ich denke, jeder Bra-ket mag X B , T B | X A , T A hat Abmessungen von 1/L. Wenn ja, dann A B | X B , T B | X A , T A | 2 D X ist keine Wahrscheinlichkeit.
@AliEsquembreKucukalic Eine Sache, die mir immer wieder durch den Kopf geht, ist, dass es eigentlich ein doppeltes Integral sein sollte, aber ich bin mir nicht ganz sicher, wie ich es rechtfertigen soll.
@InertialObserver Nein, sollte es nicht sein. Weil die Eigenzustände des Positionsoperators nicht normierbar sind.
@DvijMankad ich stimme zu

Ihre Berechnungen sind vollkommen korrekt – daher werde ich direkt auf das grundlegende Problem eingehen, das Sie ansprechen. Insbesondere warum ist die Dimensionalität von X B , T B | X A , T A nicht so, dass sein Modulusquadrat die Dimensionalität einer linearen Wahrscheinlichkeitsdichte hat, dh L 1 . Das Problem ist nicht empfindlich gegenüber dem Bild, das wir verwenden, daher werde ich das Bild von Schrödinger verwenden.

Der Grund dafür ist, dass Positionseigenzustände in einem kontinuierlichen Raum nicht normierbar sind und Dirac-normiert sind. Die Positionseigenzustände sind normiert als X | j = δ ( X j ) . Genauer gesagt, wenn ein Zustand an einer Position lokalisiert ist X 0 , seine Wellenfunktion ist per Definition ψ ( X ) = X | ψ = X | X 0 = δ ( X X 0 ) . Somit ist die Dimensionalität der Wellenfunktion selbst die einer gewöhnlichen linearen Wahrscheinlichkeitsdichte. Dies ist gerade deshalb keine Inkonsistenz, weil eine solche Wellenfunktion als nicht normierbar erkannt wurde und als Dirac-normierte Wellenfunktion definiert ist.

Natürlich sind die übliche Normalisierung und die Dirac-Normalisierung nicht dasselbe und sollten nicht nur als zwei verschiedene Arten von im Wesentlichen derselben Sache betrachtet werden. Die Tatsache, dass die Ortseigenzustände Dirac-normalisiert sind, prägt (und ist wesentlich für) ihre Beziehung zu den gewöhnlichen normierbaren Zuständen. Ein gewöhnlicher normalisierbarer Zustand, wenn er als lineare Kombination anderer derartiger Zustände ausgedrückt wird, nimmt die lineare Kombination die Form einer Summierung an. Wenn dagegen ein üblicher normierbarer Zustand als lineare Kombination von Positionseigenzuständen ausgedrückt wird, nimmt die lineare Kombination die Form einer Integration an. Hier kommt das Integrationsmaß ausnahmslos mit einer eigenen Dimensionalität, und die Dimensionalität der Positionseigenzustände wird wesentlich, um die Dimensionalität der linearen Kombination gleich der Dimensionalität des gewöhnlichen normierbaren Zustands zu machen. Beachten Sie, dass all dieses Geschäft auseinanderfallen würde, wenn die Positionseigenzustände nicht Dirac-normalisiert worden wären - da der vollständige Satz der Positionseigenzustände kontinuierlich ist, müsste ein generischer normalisierbarer Zustand als Integration verschiedener Positionseigenzustände ausgedrückt werden (und nicht als übliche Summierung von ihnen) und somit würde die Dimensionalität des Integrationsmaßes die Gleichungen ohne die "ungewöhnliche" Dimensionalität der Positionseigenzustände (die sie ihrer Dirac-Normierung verdanken) inkonsistent machen.

Schließlich ist die direkte probabilistische Interpretation eines nicht normierbaren Zustands als solche verboten. Sie sind dennoch wichtig, da normierbare Zustände als Linearkombination solcher nicht normierbarer Zustände dargestellt werden können. Siehe: https://ocw.mit.edu/courses/physics/8-05-quantum-physics-ii-fall-2013/lecture-notes/MIT8_05F13_Chap_01.pdf (Abschnitt 2 ). Man kann oft bestimmte clevere Denkweisen über probabilistische Interpretationen von nicht normalisierbaren Zuständen erfinden – oft indem man den nicht normalisierbaren Zustand als eine Grenze eines normalisierbaren Zustands betrachtet. Beispielsweise kann das Dirac-Delta als Grenze einer Gauß-Funktion angesehen werden. In ähnlicher Weise werden solche nicht normierbaren Impuls-Eigenzustände oft als die Grenze eines normierbaren diskreten Satzes von Impuls-Eigenzuständen auf einem Gitter usw. angesehen.

Gute Antwort. Wenn jedoch nicht normalisierbare Zustände nicht direkt mit einer Wahrscheinlichkeit verstanden werden können, dann. Warum funktionieren Feynmans Pfadintegrale? Feynmans Pfadintegralgleichung gibt den Propagator an, der (vielleicht ist hier mein Fehler), da es eine Amplitude ist, als Wahrscheinlichkeit verstanden werden sollte, wenn er quadriert wird.
@AliEsquembreKucukalic Ja, der Propagator ist technisch gesehen die Wahrscheinlichkeitsamplitude. Genauso wie es die positionsbasierte Wellenfunktion eines nicht normierbaren Zustands ist. Aber physikalisch gesehen ähnelt der Propagator eher der Funktion von Green. Wenn es als „Kern“ zusammen mit einem physikalischen Anfangszustand (d. h. einem normierbaren Anfangszustand) integriert würde, würde es die physikalische Wahrscheinlichkeitsamplitude ergeben, das Teilchen an einer bestimmten Position zu finden – oder äquivalent die zeitlich entwickelte physikalische Welle – Funktion.
Auch wenn ja, da der Kernel K ( X B , T B ; X A , T A ) = X B , T B | U ( T B , T A ) | X A , T A so dass ψ ( X B , T B ) = K ( X B , T B ; X C , T C ) ψ ( X C , T C ) D X C , dann haben wir wieder das Dimensionalitätsproblem beim Quadrieren ψ ( X B , T B ) .
@AliEsquembreKucukalic Genau nicht, weil der Kernel K hat die exakt umgekehrte Dimensionalität, um die Dimensionalität des Integrationsmaßes aufzuheben D X . Dies ist natürlich nur eine Wiederholung der Tatsache, dass Positions-Eigenzustände Dirac-normalisiert sind. Um es explizit zu machen, da U ^ dimensionslos ist (da es sich nur um eine Exponentialfunktion handelt), ist die Dimensionalität des Kerns die von X B | X C = δ ( X B X C ) L 1 . Also die normierbare Wellenfunktion auf der linken Seite, ψ B hat die gleiche Dimensionalität wie die normierbare Wellenfunktion auf der RHS, ψ C .